PEDS NEURO

Pataasin ang iyong marka sa homework at exams ngayon gamit ang Quizwiz!

What treatment is indicated for infant with botulism?

Botulism antitoxin (IgG)

What is the most common cause of subdural hematomas in infants?

Child abuse

Classic EEG finding in absence seizures

3-Hz generalized spike and wave pattern

Classic EEG finding in JME?

4-6 Hz polyspike and wave pattern is seen in juvenile myoclonic epilepsy, which presents in adolescence with myoclonic and/or generalized tonic-clonic seizures.

How are tics treated

The 2 medications approved by the Food and Drug Administration for use in tics are haloperidol and pimozide. The side effects of these medications often outweigh the benefit. Many providers first try an α2-adrenergic agonist, such as clonidine or guanfacine, but this is an off-label use. Another treatment for tics is comprehensive behavioral intervention for tics (C-BIT), a behavioral technique or therapy taught by psychologists or other behavioral health specialists that has been shown to reduce tic severity. The girl in the vignette is not having academic or social problems associated with her tics, so she does not need treatment.

What is the differential for acute ischemic stroke in pediatrics?

The causes for AIS in children include embolism, in situ thrombosis, vasculopathy, genetic disorder, and metabolic stroke. Embolic stroke can arise from artery-to-artery embolism resulting from a carotid or vertebral dissection; cardiogenic emboli from intracardiac thrombi; or paradoxical embolism from venous thrombosis embolizing to the arterial circulation through a patent foramen ovale, ventricular septal defect, other cardiac shunt, or through a pulmonary arteriovenous malformation. In situ thrombosis can arise in the setting of a hypercoagulable state such as factor V Leiden deficiency, prothrombin gene mutation, or an inflammatory disorder such as systemic lupus erythematosus or meningitis. Focal cerebral arteriopathy is thought to be a postinfectious process, causing transient narrowing of a cerebral artery, usually the middle cerebral artery. Fabry disease, ɑ-galactosidase A deficiency, is an X-linked disorder that presents with pain crises in childhood and adolescence, angiokeratomas, anhidrosis, and stroke in young adulthood. Causes of metabolic stroke include mitochondrial encephalomyopathy, lactic acidosis, and strokelike episodes (MELAS). In MELAS, the area of acute ischemia is in a nonvascular distribution, typically in the posterior parietal and occipital lobes.

A 13-year-old boy presents to the emergency department with complaints of blurred vision, difficulty swallowing, and vomiting. The symptoms started while the patient was at a local scout camp and have been progressing rapidly over the last several hours. On examination, the patient has dysconjugate gaze, difficulty swallowing, and proximal muscle weakness. Muscle mass is normal. Reflexes are intact. Which of the following diagnoses most closely aligns with this presentation? A Rabies B Guillain-Barré syndrome C Lyme disease D Tetanus E Botulism

The classic form of botulism typically seen in adults, but only occasionally seen in children and adolescents, is caused by the contamination of commercial food products with botulinum toxin. Microgram amounts of toxin can produce severe paralysis and interruption of autonomic nervous system function. Inadequate cooking of foods during the canning process allows the Clostridium botulinum spores to germinate, and the toxin is produced within the closed container. The syndrome develops 12-36 hours after ingestion of the toxin-containing food. Early symptoms are blurred vision, diplopia, dizziness, dysarthria, and dysphagia and are followed by a descending paralysis and dyspnea. Electrophysiologically, incremental response on repetitive nerve stimulation, and small compound motor action potentials are usually present. Single-fiber electromyography may be necessary to demonstrate increased jitter and impulse blocking. Respiratory paralysis and death may develop. With supportive care, recovery is expected within weeks to months. Equine-derived antitoxin is used in children ≥ 1 year of age and in adults. Guillain-Barré syndrome (GBS) is an ascending flaccid paralysis that usually follows an antecedent illness, typically an upper respiratory illness or gastroenteritis. Paresthesias are often the first symptoms, followed by weakness. The course usually progresses for up to 4 weeks and then resolves over the next 2-4 weeks. Areflexia or hyporeflexia is a cardinal feature and is seen at presentation. Rabies is a viral encephalitis that is generally fatal if not treated before the central nervous system is infected. The initial indication of infection is a general malaise with flu-like symptoms. This is followed by confusion and irritability, which are accompanied by a descending paralysis that affects the muscles of swallowing. Progression of the encephalitis leads to coma, and respiratory compromise is the eventual cause of death. Tetanus causes profound muscle spasms in affected individuals. The initial symptom tends to be spasm of the jaw muscles, leading to the term "lockjaw." The spasms advance in a descending pattern, with neck and respiratory muscles being followed by generalized muscle spasm. Neuropathy associated with Lyme disease occurs weeks to years after infection with Borrelia burgdorferi and is primarily sensory. The distribution is usually patchy and asymmetric.

Do febrile seizures require treatment?

A brief (< 15 minutes) generalized seizure soon after developing an increase in temperature, without any abnormal neurologic findings on physical examination to suggest a central nervous system infection or disorder. In addition, he has no record of developmental delay or chronic neurologic disorder. Such a history in a child between 6 months and 5 years of age is consistent with a simple febrile seizure, and no antiseizure medications are indicated. The majority of febrile seizures occur between 12 and 18 months of age, most often when the temperature is rapidly rising during the 1st day of illness. Although the overall recurrence rate is 30-35%, the risk of recurrence is much higher (60-65%) in children < 1 year of age at the time of the 1st seizure. The risk in older children is lower (20%). In addition to age at onset of 1st seizure, the recurrence risk is also higher if there is a history of a 1st degree relative with febrile seizures. Neurologic sequelae are very rare, and the risk of subsequent epilepsy in children with febrile seizures is only slightly above that of the general population. In general, the risks of continuous or intermittent treatment with an antiseizure medications outweigh any potential benefits of reducing the risk of recurrent febrile seizures. Additionally, there is no evidence to suggest that treatment to prevent febrile seizures will lessen the risk of subsequent epilepsy. For children with a previous history of febrile seizures, treatment with antipyretics during a febrile illness may be helpful in the general management but does not affect the recurrence rate of febrile seizures.

What is first line treatment for infantile spasms?

Adrenocorticotropic hormone (ACTH; a.k.a. corticotropin) and vigabatrin are considered 1st line treatments for infantile spasms. In patients with infantile spasms due to tuberous sclerosis, vigabatrin is the drug of choice. A variety of antiseizure drugs have been used in affected children who do not respond to ACTH or vigabatrin—but usually with only partial, if any, improvement in symptoms. Pyridoxine (vitamin B6) is sometimes effective.

what brain findings are expected in a pediatric patient with spina bifida ( a sac protruding from the lumbar spine with fluid and spinal cord contents)?

An enlarged head circumference, myelomeningocele, and club feet, are all symptoms of spina bifida. Given these findings, one would expect to see hydrocephalus on MRI of the brain (as a consequence of Chiari Type II [a.k.a. Arnold-Chiari] malformation). Chiari Type II is downward displacement of the cerebellar vermis, the fourth ventricle, and the medulla below the foramen magnum. This leads to obstruction of cerebrospinal fluid flow and to noncommunicating hydrocephalus. Chiari Type II malformation is commonly associated with myelomeningocele, but Chiari Type I malformation is less likely. Chiari Type I is the downward displacement of the cerebellar tonsils, and in severe cases the medulla, through the foramen magnum.

A 16-year-old male comes to the clinic after a period of several "no shows" for follow-up of his seizure disorder. He has been on carbamazepine for several years because of focal seizures with impaired consciousness (formerly known as complex partial seizures). Because he has not been seen in several years, his labs are checked and reveal a carbamazepine level of 3.1 µg/mL (normal 4-12 µg/mL). When questioned, the patient acknowledges that he has not had a seizure now for the past 8 months since he has been taking his medication faithfully. (He previously was more erratic and thus would get breakthrough seizures when missing medication doses.) He is tolerating his dosing regimen of 400 mg bid without side effects. His MRI brain scan at initial presentation was normal, and prior EEG showed some interictal spikes in the left temporal lobe. Based on this information, what is the best recommendation at this point? A Continue carbamazepine at the current dose. B Get a repeat EEG, and then decide about dose adjustment. C Discontinue carbamazepine. D Increase the dose of carbamazepine by 200 mg a week up to 600 mg bid to get to a more therapeutic level for better seizure prevention. E Add a 2nd antiseizure medication.

Answer A Continue carbamazepine at the current dose. Explanation Antiseizure medications are employed to control seizure activity. Therapeutic ranges are established as guidelines, which represent serum levels that are typically necessary to control seizures. The upper limits also serve as an aid to reduce symptomatic side effects. Serum drug levels should not be used in isolation to guide therapy as the therapeutic range is different for individual patients. The benefit from seizure control is balanced with the difficulties arising from side effects. In this case, the patient is having no side effects and does have very good seizure control when taking this dose (being seizure free for the past 8 months when taking the medication faithfully). Adding a 2nd agent or increasing the dose is not necessary because the seizures are adequately controlled at this point despite the drug level being a little low.

The parents of a 10-year-old girl report that their daughter's school performance has not improved since beginning methylphenidate for "attention-deficit disorder." Her teacher and her parents have noted that she often stares ahead and it seems as though she is daydreaming. During examination, she is made to hyperventilate by blowing a pinwheel. Soon afterwards she is noted to have an 8- to 10-second episode of staring while blinking her eyes and opening/closing her mouth. An EEG shows 3-Hz spike and wave discharges. Which of the following is the best initial treatment? A Ethosuximide B Gabapentin C Phenobarbital D Carbamazepine E Phenytoin

Answer A Ethosuximide Explanation This patient has childhood absence epilepsy, a form of primary generalized epilepsy. Ethosuximide is considered 1st line therapy. A history of repetitive, brief episodes of staring along with altered awareness, eyelid movements, and/or one or more automatisms are typical findings in patients with childhood absence epilepsy. Although children may experience multiple seizures throughout the day, they often go unrecognized or are misinterpreted as inattention. Patients have no associated aura or postictal phase; hyperventilation is an effective trigger in the majority of children. Children are not responsive during the seizure and have no memory of the event. The EEG typically shows regular and symmetrical 3-Hz spike and slow wave complexes.

During a health maintenance visit, a 5-month-old boy is observed to have several clusters of episodes of seizure-like activity. He is in the 20th percentile for height and weight and the 5th percentile for head circumference. His physical examination is positive for mild generalized hypotonia. He also has difficulty rolling from front to back, and there remains appreciable head lag when he is lifted from his back while grasping his hands. An EEG is ordered, which is read as "very high-voltage, random, disorganized, slow waves, and spikes that vary in duration and are located in all cortical areas." Which of the following is most likely to describe the clinical characteristics of this patient's seizure activity? A Flexor spasms of the neck, arms, and legs followed by sudden extension of the neck, arms, trunk, and legs B Unilateral tonic-clonic activity associated with unresponsiveness and a prolonged postictal state C Horizontal nystagmus associated with torticollis and head bobbing D Prolonged episodes of generalized hypotonia, often associated with cyanosis E Lip smacking, tongue thrusting, and vertical nystagmus associated with generalized hypertonia

Answer A Flexor spasms of the neck, arms, and legs followed by sudden extension of the neck, arms, trunk, and legs Explanation The patient has EEG findings consistent with hypsarrythmia—high-voltage, irregular, slow waves that occur out of sync and randomly over all head regions are intermixed with spikes from multiple foci. Hypsarrythmia is the EEG pattern in infantile spasms, a seizure disorder that usually presents within the 1st year of life, characterized by clusters of muscle spasms (classically flexor spasms described as sudden, simultaneous flexion of the neck and trunk, with flexion and adduction of the extremities). Spasms typically last only seconds and occur in clusters of several to > 100. The long-term prognosis for normal neurocognitive development is poor. The 2 most efficacious treatments for infantile spasms include intramuscular adrenocorticotropic hormone (ACTH) and oral vigabatrin with close monitoring for improvement on EEG. Infantile spasms may be associated with tuberous sclerosis. Therefore, patients with infantile spasms should be closely evaluated for clinical evidence of this disorder, including examination of the skin with the aid of a Wood lamp to identify ash-leaf spots. Vigabatrin is the drug of choice for use in infantile spasms with coexistent tuberous sclerosis.

A 3-year-old boy presents for his first health maintenance visit after arriving in the United States from an orphanage in the Dominican Republic. His adoptive parents know little about his past medical history, except that the patient was abandoned by his natural mother soon after birth. On physical examination, his neck appears shortened and broad, and he has a low occipital hairline. A plain radiograph shows evidence of fusion and hypoplasia of the C3-C7 vertebrae. Which of the following is most likely to be identified during additional evaluation of this patient? A Hypoplastic scapula B Syndactyly C Hemihypertrophy D Pseudarthrosis of the clavicle E Absent radii

Answer A Hypoplastic scapula Explanation The patient has clinical signs and symptoms consistent with Klippel-Feil syndrome, characterized by failure of segmentation and fusion of ≥ 2 cervical vertebrae. This syndrome is often associated with Sprengel deformity, in which one or both scapulae are abnormally small and high-riding. Affected individuals are unable to raise their arm(s) fully.

The parents of an 11-year-old girl with a noticeable speech impediment express concern that their daughter is having increasing difficulty climbing stairs and has become "more clumsy." On physical examination, wasting is evident in the muscles of the dorsal forearm and anterior compartment of the lower legs. When asked, the patient has difficulty quickly opening her hands from a fisted position. Percussion of the thenar eminence results in opposition of the thumb and a delayed relaxation phase. A serum creatine phosphokinase (CPK) level is within the normal range. Which of the following is the most likely diagnosis in this patient? A Myotonic dystrophy B Emery-Dreifuss muscular dystrophy C Becker muscular dystrophy D Charcot-Marie-Tooth disease E Duchenne muscular dystrophy

Answer A Myotonic dystrophy Explanation Myotonic dystrophy is a multisystem disorder transmitted by autosomal dominant inheritance with variable penetrance. A neonatal form occurs in children born to mothers with myotonic dystrophy. The onset of symptoms typically occurs during adolescence or later. The major features are myotonia (a disturbance in muscle relaxation after contraction), weakness in the face and distal portion of the limbs, cataracts, frontal baldness, and multiple endocrinopathies. The pattern of muscle atrophy in the face is so stereotypical that all patients with the disease have a similar facies. The face is long and thin because of wasting of the temporal and masseter muscles, and the neck is thin because of atrophy of the sternocleidomastoid muscles. The eyelids and corners of the mouth droop, and the lower part of the face sags, producing the appearance of sadness. Percussion of muscle demonstrates myotonia. When percussed, the thenar eminence dimples and remains dimpled at the site of percussion. In addition, the thumb abducts and remains in that position for several seconds. The physician can also detect myotonia by shaking hands with the patient, who has difficulty letting go and releases the grip in part by flexing the wrist to force the finger flexors to open. Smooth and cardiac muscle involvement may be present. Disturbed gastrointestinal motility is characteristic. Endocrine disturbances include testicular atrophy, infertility in women, hyperinsulinism, adrenal atrophy, and disturbances in growth hormone secretion.

A 17-year-old boy who is currently being treated with several medications for bipolar disorder is hospitalized after cutting himself to "relieve the stress" so that he "could sleep after partying for 3 days." During routine laboratory screening upon admission, he is noted be hyponatremic, with a serum Na+ level of 129 mmol/L. Which of the following medications is most often associated with this finding? A Oxcarbazepine B Levetiracetam C Gabapentin D Valproic acid E Phenytoin

Answer A Oxcarbazepine Explanation Oxcarbazepine (Trileptal, Oxtellar XR) may be associated with clinically significant hyponatremia, usually within the first 3 months of therapy. The extent of hyponatremia is dose related and most often asymptomatic. It is generally not necessary to reduce or discontinue oxcarbazepine because serum Na+ levels typically return to normal or stabilize at a mildly reduced level. In symptomatic patients or in those with Na+ levels < 120 mEq/L, a reduction in dose of oxcarbazepine and/or mild fluid restriction usually returns Na+ to an acceptable level. Additional adverse effects include headache, dizziness, lethargy, nystagmus, diplopia, ataxia, and nausea. Although considered an antiseizure drug, oxcarbazepine is among several antiseizure medications sometimes prescribed in patients with mood disorders. As many as 25-30% of patients with a history of hypersensitivity reactions to carbamazepine experience a cross-reaction to oxcarbazepine. Concomitant polytherapy with carbamazepine, phenobarbital, valproic acid, or phenytoin may decrease levels of oxcarbazepine. Oxcarbazepine may also decrease the effectiveness of oral contraceptives in females. None of the other medications are associated with clinically significant hyponatremia. Phenytoin usage is associated with gingival hypertrophy, rash, ataxia, and agranulocytosis. Levetiracetam can be associated with behavior changes like emotionality and irritability. Gabapentin is most typically associated with dizziness and somnolence. Valproic acid is associated with alopecia, dizziness, hyperammonemia, tremor, and weight gain.

A 3-day-old term infant is transferred to the neonatal intensive care unit after a tonic seizure associated with a period of apnea and cyanosis. She continues to have recurrent seizures over the next 24 hours despite treatment with benzodiazepine, phenytoin, and phenobarbital. Which of the following nutrient deficiencies has a well-known association with neonatal seizures that are particularly resistant to treatment with conventional antiseizure therapy? A Vitamin B6 B Vitamin C C Vitamin K D Vitamin D E Vitamin B2

Answer A Vitamin B6 Explanation Seizures in neonates who do not respond to conventional antiseizure therapy may be due to pyridoxine (vitamin B6) deficiency. Pyridoxine is required for the synthesis of γ-aminobutyric acid (GABA), the principle inhibitory neurotransmitter in the central nervous system. Affected infants require large amounts of supplemental pyridoxine to maintain adequate production of GABA. Intravenous administration of pyridoxine can result in improvement of the EEG in pyridoxine deficiency or dependence. Riboflavin (B2) deficiency is associated with cheilosis and sore tongue. Vitamin K (phylloquinone) is necessary for maintaining prothrombin, Factor 7, Factor 9, and Factor 10; hence, vitamin K deficiency causes blood clotting and bleeding issues. Generally, vitamin K deficiency is most common in children (particularly newborns and infants) who have not yet developed a significant bacterial gastrointestinal flora. Vitamin D is necessary for bone growth and maturation. If it is not available, rickets can occur in children. Initial symptoms of vitamin C deficiency (a.k.a. scurvy) are nonspecific and include irritability, digestive disturbances, and anorexia. Classic physical descriptions include follicular hyperkeratosis and "corkscrew-coiled" hairs. Gingival bleeding is also common. Normochromic, normocytic anemia is found in ~ 75% of patients with scurvy.

A 13-year-old child comes in with a history of several weeks of increasing headaches and vomiting. He is afebrile and has no history of infection. His symptoms worsen with lying flat. He complains of double vision. Which of the following cranial nerve abnormalities might you discover associated with this disease? A 4th nerve palsy B 6th nerve palsy C 7th nerve palsy D 10th nerve palsy E 8th nerve palsy

Answer B 6th nerve palsy Explanation A 6th nerve palsy, often called a "false localizing sign," is very frequently seen in children with increased intracranial pressure (ICP). The child in the clinical scenario has some cardinal symptoms suggestive of increased ICP, including increasing tempo of headaches, together with vomiting, diplopia, and worsening with positional changes. His funduscopic examination may reveal papilledema. The patient should have urgent neuroimaging with MRI of the brain to evaluate for the presence of a space-occupying lesion. Frequently, masses in the posterior fossa will result in this clinical scenario. Another important consideration is sinovenous thrombosis. In the absence of abnormalities on neuroimaging and the presence of papilledema and headaches, consider the diagnosis of pseudotumor cerebri. To confirm the diagnosis, perform a lumbar puncture with opening pressure in the lateral decubitus position.

A 2-year-old girl is transported to the emergency department after her parents noted that she was unable to walk after awakening from a nap. Just prior to arrival in the emergency department, she vomited twice. She is afebrile. On physical examination, she is awake and alert, but she is unable to stand or sit without support because of severe truncal ataxia. Additional clinical findings include horizontal nystagmus and difficulty with speech. An emergent MRI scan of the head is normal, followed by a lumbar puncture, which reveals 1 red blood cell, 15 lymphocytes, a glucose level equal to 70% of the serum glucose, and a protein level of 15 mg/dL. A stat EEG shows a normal background and no epileptiform activity. Urine drug screen is negative. On further questioning, her parents reveal that their daughter had a rash 3 weeks prior to presentation, which was described as generalized pruritic, maculopapular, and vesicular lesions, which became crusted within several days, while, at the same time, new vesicular lesions appeared. What is the most likely diagnosis? A Subclinical seizure B Acute cerebellar ataxia C Acute drug ingestion D Acute head trauma E Opsoclonus-myoclonus-ataxia syndrome

Answer B Acute cerebellar ataxia Explanation The patient has clinical findings of ataxia associated with a normal head MRI, negative urine drug screen, and normal cerebrospinal fluid findings with the exception of very mild pleocytosis. Such a scenario is consistent with acute cerebellar ataxia of childhood, a diagnosis of exclusion typically seen among young children between 1 and 3 years of age (though it can also be seen up to ∼ 6 years of age). The antecedent rash history describes the typical rash of varicella. Acute cerebellar ataxia most commonly follows certain viral infections, especially varicella, echovirus, and coxsackievirus infections, and is likely the result of an autoimmune mechanism. Associated findings include truncal ataxia, horizontal nystagmus, difficulty with fine motor control, tremors, and dysarthria. Prior to diagnosing acute cerebellar ataxia, patients require detailed evaluation to exclude prior recent trauma, toxic ingestion, a central nervous system mass, neuroblastoma, and meningitis or encephalitis. Most patients improve with symptomatic treatment within several weeks, but a minority remain symptomatic for up to several months.

as pounding, moderate in intensity, and located in the bilateral frontal area. She has no aura and frequently experiences nausea and vomiting. There is associated photophobia and phonophobia. Which of the patient's symptoms is more frequently seen in children who suffer from migraine headaches, as compared to adult sufferers? A Photophobia B Bilateral pain C Phonophobia D Presence of an aura E Osmophobia

Answer B Bilateral pain Explanation Pediatric migraine headaches are more likely to be bilateral, rather than unilateral, as is usually seen in adults. The pain can often be of a shorter duration (less than 4 hours). Other common associated symptoms may include dizziness, pallor, nausea, vomiting, and sensitivity to external stimuli such as light (photophobia) and noise (phonophobia). The most common migraine-type headache in children is migraine without aura. Chemical triggers may be present but are less commonly seen in children. These include monosodium glutamate (MSG), aspartame, tyramine, and phenylethylamine. Several periodic conditions are thought to be precursors of migraines. These include cyclic vomiting syndrome, characterized by repeated episodes of nausea and vomiting lasting hours to days, separated by symptom-free periods of varying duration; abdominal migraine, which may occur in association with cyclic vomiting, characterized by abdominal pain, nausea, vomiting, and pallor; and benign paroxysmal vertigo of childhood, associated with short episodes of disequilibrium, vertigo, nausea, and nystagmus. Migraines with aura are less frequent in the pediatric population. Osmophobia is not a part of the diagnostic criteria for migraines. Photophobia and phonophobia can be seen in pediatric as well as adult migraines.

The parents of an 8-year-old child bring their son to the clinic with complaints of clumsiness and frequent falls. This has been present for several years but initially was very mild. Over the last several months, they have noted a significant worsening. Social history is remarkable for the fact that the child was adopted at 2 years of age, and the parents have no information about the birth family. On examination, the child does have an awkward gait. There is moderate atrophy of the distal muscles of the lower extremities. The patient also has high-arched feet. Deep tendon reflexes are absent in the lower extremities and are equivocal in the upper extremities. Which of the following diagnoses is most consistent with this presentation? A Guillain-Barré syndrome B Charcot-Marie-Tooth disease C Duchenne muscular dystrophy D Mononeuritis multiplex E Becker muscular dystrophy

Answer B Charcot-Marie-Tooth disease Explanation Charcot-Marie-Tooth (CMT) disease and hereditary motor sensory neuropathy are interchangeable terms. It is caused by a variety of gene defects that share a common clinical manifestation. Symptoms typically present in the 1st or 2nd decade, and a family history is common. The typical patient has progressive distal weakness, mild to moderate sensory loss, depressed or no tendon reflexes, and high-arched feet (pes cavus). Muscle atrophy develops and leads to "stork legs." Upper extremities are eventually involved in later stages of the disease process

A CT scan in a 3-year-old boy reveals clouding of the left middle ear and mastoid, loss of definition of the bony septae, coalescence of the mastoid air cells, and irregularity of the mastoid cortex. Associated clinical findings in this patient are most likely to include which of the following? A Proptosis B Displacement of the auricle C Erythema, warmth, and tenderness overlying the preauricular area D Purulent, foul-smelling discharge from the left nares E Softening and thinning of the skull (craniotabes) above and anterior to the auricle

Answer B Displacement of the auricle Explanation CT findings described in the vignette are typical of acute coalescent mastoiditis, defined as inflammation of the mastoid air cells associated with spread of the infection to the periosteum (periosteitis) and bony trabeculae of the petrous bone (osteitis). Associated clinical signs and symptoms include moderate-to-severe ear pain (otalgia) and postauricular tenderness, erythema, and swelling. Displacement of the auricle is typical—usually downward and outward in children < 2 years of age, and upward and outward in those 2 years of age and older. Clinical manifestations of acute otitis media are noted in the majority of patients at the time of the diagnosis; additional findings may include fever, malaise, irritability, narrowing of the external ear canal, and perforation of the tympanic membrane with or without otorrhea. Extracranial complications include facial nerve palsy, subperiosteal abscess, osteomyelitis, and transient or permanent hearing loss; intracranial complications may include meningitis, abscess, and venous sinus thrombosis. The most common causes of acute mastoiditis include Streptococcus pneumoniae, nontypable Haemophilus influenzae, Streptococcus pyogenes, Staphylococcus aureus and Pseudomonas aeruginosa. Treatment includes intravenous antibiotics and myringotomy; mastoidectomy is indicated in more extensive disease.

The parents of a 5-year-old boy complain that their son is no longer able to keep up with his peers while playing in the playground and often complains of "feeling tired," even after minimal physical activity. On physical examination, enlargement of the calf muscles and thigh muscle wasting are noted. His deep tendon reflexes in the lower extremities are normal, despite bilateral lower-extremity weakness and an unsteady gait. Laboratory values include elevated serum transaminase levels. Which of the following is the most likely diagnosis in this patient? A Dermatomyositis B Duchenne muscular dystrophy C Pompe disease D Myasthenia gravis E Spinal muscular atrophy Type 3

Answer B Duchenne muscular dystrophy Explanation The patient has clinical findings consistent with Duchenne muscular dystrophy (DMD), the most common hereditary neuromuscular disorder affecting boys and the most common form of muscular dystrophy. Inherited in an X-linked recessive pattern, the disorder is caused by a defect in the dystrophin gene located on the X chromosome at Xp21, leading to muscle fiber degeneration in the skeletal and cardiac muscle. Lower extremity proximal muscle weakness, which is typically observed prior to upper extremity findings, results in hip girdle weakness and an abnormal gait. Gowers sign (the patient uses hands and arms to "walk" up the body from a squatting position; see image), toe walking, and hip weakness are characteristic findings, which may be observed as early as 3-4 years of age. Pseudohypertrophy (replacement of muscle by fat and other connective tissue giving the appearance of hypertrophy) of the calf and gluteal muscles is associated with thigh muscle wasting; as the disease progresses, deltoid muscles are also affected. Joint contractures and scoliosis often confine the patient to a wheelchair by the second decade of life. Cardiomyopathy, recurrent respiratory infections, and respiratory insufficiency often lead to death by early adulthood. Patients with DMD are likely to have academic difficulties due to underlying cognitive impairment. Mean IQ score in one study was reported to be 83.

During the well-child visit for an 11-year-old boy, you note that he has developed increased proximal weakness of the lower extremities, some gait unsteadiness, decreased visual acuity, and displacement of the point of maximal impulse (PMI) on palpation of the chest. He has also developed mild scoliosis and an increase in the arch of both feet. His muscle mass appears to be normal for his age; he has diminished deep tendon reflexes but bilateral Babinski sign. His parents have noted that he has decreased his physical activities over the preceding 2 years and no longer plays with his friends as he previously did. There is no family history of a similar disorder, and his 2 older siblings appear well. With which of the following disorders is his presentation most consistent? A Chronic inflammatory demyelinating polyradiculoneuropathy B Friedreich ataxia C Charcot-Marie-Tooth disease D Duchenne muscular dystrophy E Juvenile-onset spinal muscular atrophy

Answer B Friedreich ataxia Explanation Friedreich ataxia is an autosomal recessive degenerative neurologic condition that manifests with a spectrum of pathology that includes ataxia, lower extremity weakness, cardiomyopathy, eye and ear difficulties, hypoactive or absent deep tendon reflexes, and bony abnormalities (pes cavus and scoliosis). There is also a very high prevalence (20%) of diabetes in these individuals. Juvenile spinal muscular atrophy presents between 2 and 17 years of age with proximal muscle weakness. Pyramidal signs are absent (the patient in the vignette has Babinski sign) and there is no involvement of the visual or cardiac systems. Duchenne muscular dystrophy is an X-linked recessive myopathy that predominantly affects proximal muscle groups. Affected individuals are typically wheelchair-bound by 12 years of age and die of respiratory failure in their teens to early 20s. Charcot-Marie-Tooth disease is an inherited neuropathy in which patients demonstrate progressive distal weakness that is ascending in nature. Muscle atrophy develops and leads to "stork legs," and deep tendon reflexes are lost. Upper extremities are eventually involved in later stages of the disease process. Chronic inflammatory demyelinating polyradiculoneuropathy (CIDP) is characterized by fluctuating course, distal symmetric muscle weakness, and hypo- or areflexia.

A 4-year-old boy with trisomy 21 presents to the emergency department following his 3rd seizure in the last 4 months. He appears to be in a postictal state, and his initial neurological exam is concerning for lack of withdrawal of the left upper extremity upon painful stimulation. After 48 hours, he is back to his baseline, except that he is still unable to move his left upper extremity. There is no history of trauma, and plain films of the extremity are normal. A cranial CT shows a hypodensity in the right cerebral hemisphere. Additional imaging in this patient is most likely to reveal which of the following disease processes? A Intracranial abscess B Moyamoya disease C Atlantoaxial subluxation D Right epidural hematoma E Intracranial aneurysm

Answer B Moyamoya disease Explanation The hypodensity seen on CT and the patient's exam is suggestive of an ischemic infarct or stroke. Of the provided answers, moyamoya is the most likely to cause an infarct. Moyamoya disease is a chronic, progressive cerebrovascular disorder that can present in childhood. "Moyamoya" is a Japanese word meaning "puff of smoke;" its cause is unknown but can be seen as a primary disease or a secondary vasculopathy associated sickle cell disease, trisomy 21, collagen vascular disorders, and neurofibromatosis Type 1. The "puff of smoke" sign, seen on cerebral angiography, represents a network of abnormally dilated collateral vessels, formed in response to progressive unilateral or bilateral stenosis in the terminal portions of the internal carotid arteries and their main branches. Clinically, moyamoya disease may present with a stroke, transient ischemic attack, a history of chronic headaches, or with new onset of seizures. Hyperventilation may lead to vasoconstriction of already compromised cerebral circulation and may precipitate a transient ischemic attack. Treatment is aimed at improving cerebral blood flow through neurosurgical revascularization.

Soon after awakening from a nap, a 5-year-old girl is rushed to the emergency department after her grandparents noticed that she had great difficulty walking and had fallen several times, "like she could not keep her balance." Her vital signs are stable, and she is afebrile. Although her speech is slurred, she appears alert and oriented without associated nuchal rigidity. Positive findings on physical exam include horizontal nystagmus. A urine drug screen is negative. Examination of the cerebrospinal fluid and an MRI of the head are normal. Which of the following best describes physical findings that often precede the onset of symptoms similar to those of this patient? A A monomorphous rash consisting of pale, pinkish-flesh colored papulovesicles symmetrically distributed over the extensor surfaces of the extremities, buttocks, and face B Prominent erythema of the cheeks that spares the perioral area associated with a macular erythematous "lacy" rash on the upper extremities C A pleomorphic pruritic rash consisting of macules, papules, vesicles on an erythematous base, and pustules associated with crusting of older lesions D A generalized, fine papular erythematous rash that is especially prominent in the axilla, groin, and flexor surfaces of the upper extremities E A localized erythematous papule with central puncta and well-demarcated borders

Answer C A pleomorphic pruritic rash consisting of macules, papules, vesicles on an erythematous base, and pustules associated with crusting of older lesions Explanation This patient has clinical signs and symptoms consistent with acute cerebellar ataxia, a relatively uncommon postinfectious complication of varicella. The characteristic rash associated with varicella is described as a pleomorphic pruritic rash consisting of macules, papules, vesicles on an erythematous base, and pustules associated with crusting of older lesions. Acute cerebellar ataxia has also been described following infection due to coxsackievirus, echovirus, Mycoplasma pneumoniae, and Epstein-Barr virus. The onset of truncal ataxia is sudden, generally occurring within several weeks after the onset of the precipitating infection. Associated symptoms include horizontal nystagmus, slurred speech, clumsiness with daily activities, headache, and dizziness. Symptoms generally resolve spontaneously within weeks to several months of onset. Physical therapy may hasten recovery. Although patients are often irritable and frightened, mental status is preserved throughout the illness. Most patients undergo neuroimaging at the onset of the illness to rule out a space-occupying lesion, and many undergo lumbar puncture to rule out an infectious etiology.

The parents of a 6-week-old girl comment that their daughter consistently "winks" her left eye during feeding and ask if they should be concerned. On physical examination, ptosis of the left eyelid is present; as the child sucks on a bottle, simultaneous eyelid blinking is confirmed. Which of the following best explains the etiology of these clinical findings? A Abnormal innervation of the hypoglossal and oculomotor nerves B Abnormal innervation of the trigeminal and facial nerves C Abnormal innervation of the trigeminal and oculomotor nerves D Abnormal innervation of the facial and oculomotor nerves E Abnormal innervation of the glossopharyngeal and oculomotor nerves

Answer C Abnormal innervation of the trigeminal and oculomotor nerves Explanation The patient has manifestations of the Marcus Gunn (jaw-winking) phenomenon, characterized by simultaneous eyelid blinking during sucking-jaw movements as the child contracts the pterygoid muscle. The disorder is due to abnormal innervation of the trigeminal (controlling muscles of mastication) and oculomotor nerve (controlling levator palpebrae superioris). Associated amblyopia, refractive errors, or strabismus may be seen. Although usually a benign phenomenon, surgical intervention is necessary when the associated ptosis is severe or causes amblyopia. Facial nerve abnormalities cause twitching, weakness, or paralysis of the face. The disorder may be caused by many different diseases, including circulatory disturbances, injury, infection, and tumor.

A 4-week-old male has a respiratory arrest during positioning for a lumbar puncture to rule out meningitis as the cause for lethargy, poor feeding, and decreased tone. He is quickly stabilized, and his respiratory rate returns to normal. Ampicillin, gentamicin, and acyclovir are ordered for treatment of presumed bacterial sepsis and to provide coverage for herpes simplex virus. Within 30 minutes of receiving the antimicrobial agents and acyclovir, the infant develops respiratory arrest and requires mechanical ventilation. Signs and symptoms of illness caused by which of the following is known to have the potential to worsen after antibiotic therapy? A Listeria monocytogenes B Klebsiella pneumoniae C Clostridium botulinum D Neisseria meningitidis E Group B Streptococcus

Answer C Clostridium botulinum Explanation The patient has clinical manifestations consistent with infant botulism. Infant botulism develops following ingestion of spores of Clostridium botulinum, which then germinate, multiply, and produce toxin in the intestine. In contrast to children > 1 year of age, spores can germinate and produce toxin in the intestinal tract of infants because of a relative lack of gastric acid, decreased levels of normal flora, and lack of secretory immunoglobulin A. Infant botulism is not associated with fever. Constipation, although often overlooked, is frequently the first clinical symptom of infant botulism. Poor feeding, weak cry, diminished gag and suck reflexes, hypotonia, lethargy, and excessive drooling are common. Ptosis, cranial nerve palsies, and a symmetrical flaccid paralysis, especially of the head and neck, may also occur. Shallow respirations associated with an increase in respiratory rate may lead to periods of apnea or respiratory arrest. Antibiotic therapy is not indicated in the treatment of infant botulism because bacterial cell lysis and death release the intracellular toxin into the intestinal lumen; aminoglycoside agents in particular potentiate the paralytic effects of the toxin at the neuromuscular junction. Listeria monocytogenes is a recognized, yet uncommon, cause of neonatal sepsis and meningitis. Ampicillin remains the drug of choice for treatment of Listeria infections. The course of the infection caused by Listeria, however, is not worsened by the administration of antibiotic agents. In fact, delay in the initiation of antibiotic therapy may be associated with worse outcome. Group B Streptococcus (S. agalactiae) is a well-identified cause of early and late-onset neonatal sepsis and meningitis. The prompt administration of antibiotics is a mainstay of successful management of the infected infants and is not known to worsen the course of illness. Klebsiella pneumoniae is among the Enterobacteriaceae that can cause neonatal sepsis. The most common of these organisms is Escherichia coli. Klebsiella, like many gram negative bacteria, have lipopolysaccarides (LPS) in the bacterial wall. Antibiotic treatment of gram negative sepsis may theoretically release LPS and cause septic shock; however, the clinical relevance is debatable in this scenario. The patient in the vignette develops respiratory arrest which points to a neuromuscular process, secondary to botulism and aminoglycosides. Neisseria meningitides does not commonly cause neonatal sepsis and meningitis. It is however, along with S. pneumoniae, the most common cause of meningitis beyond the neonatal period. In addition to supportive management, treatment with antibiotics can be life saving and can improve long-term outcome in infected individuals.

A 17-year-old female presents to the emergency department after her mother found her on the bathroom floor "stiff, drooling, and uncontrollably shaking her arms and legs." Her mother denies any previous history of similar episodes or known seizures. She is otherwise well with the exception of "morning jerks," which have been diagnosed as "tics" related to her use of methylphenidate for attention-deficit/hyperactivity disorder (ADHD). On further history, the patient's mother expresses concern that her daughter's "tics are sometimes so severe that she has trouble getting ready for school in the morning." The "tics" are reported to be infrequent after the morning hours. An EEG, especially upon photic stimulation, is abnormal. Which of the following describes the typical clinical course in such patients? A Resolution of all the symptoms after discontinuation of stimulant medications for ADHD B Periventricular calcifications on MRI of the brain C Effective control of subsequent seizures upon treatment with levetiracetam D Progressive cognitive and functional decline with worsening seizures leading to early death E Onset of frequent and difficult-to-control seizures despite aggressive therapy with multiple antiseizure medications

Answer C Effective control of subsequent seizures upon treatment with levetiracetam Explanation The patient has a history of "tics" followed later in adolescence by the development of a seizure disorder. Typical of patients with juvenile myoclonic epilepsy, generalized tonic-clonic seizures are often preceded by a history of frequent myoclonic jerks beginning in early- to mid-adolescence, which may be misdiagnosed as motor tics, especially in patients taking stimulant medications. The myoclonic seizures are typically seen in the early morning hours and affected individuals may have difficulty holding objects such as a toothbrush or comb. (Parents often report that the adolescent is clumsy in the morning.) Generalized convulsive seizures develop later and may be the presenting complaint in individuals who do not seek medical advice for myoclonic seizures. EEG shows presence of generalized epileptiform discharges (4-6 Hz polyspike and slow waves), which are often triggered by photic stimulation. Neuroimaging is normal and usually not performed in this condition. Valproate is the drug of choice. If valproate is contraindicated or undesirable (especially in postpubertal girls due to increased risk of neural tube defects if pregnancy occurs), levetiracetam or lamotrigine are alternative 1st line options. Response to treatment is excellent in patients compliant with daily long-term antiseizure therapy. Use of stimulant medications for treatment of ADHD may precipitate tics in some patients.

A 15-year-old boy presents with his 1st generalized tonic-clonic seizure while playing basketball with his friends. There is no history of head trauma or other provoking factors. After the seizure, he was sleepy for a couple of hours but is now back to his normal self. Several times over the last 4 weeks, his arm "twitched" while combing his hair in the morning; on 2 occasions he "twitched" and threw his comb against the wall. He reports that he was aware during each incident and did not experience any additional symptoms. Initial temperature was 99.0° F (37.2° C), and his physical examination is normal. He mentions that he is stressed because his parents are undergoing a divorce. He denies alcohol use. A routine EEG shows generalized abnormal discharges with 4-6 Hz frequency. Which of the following is the most likely diagnosis? A Psychogenic nonepileptic seizures B Absence seizures C Juvenile myoclonic epilepsy D Febrile seizures E Alcohol withdrawal seizures

Answer C Juvenile myoclonic epilepsy Explanation The patient has a history typical of juvenile myoclonic epilepsy, an epilepsy syndrome that usually begins during adolescence. Seizure activity is characterized by myoclonic seizures (uni/bilateral single or repetitive, irregular, myoclonic jerks ["twitches"] of the extremities) as well as generalized tonic-clonic seizures. Seizures are frequently seen upon awakening and exacerbated by sleep deprivation, alcohol use, psychological stressors, and photic stimulation. The EEG in untreated individuals is typically abnormal with a 4-6 Hz generalized polyspike and slow wave discharge. The exact mode of inheritance is unclear. Some patients with juvenile myoclonic epilepsy have a positive family history of epilepsy. This is a lifelong condition, and seizures are generally well controlled with levetiracetam or valproic acid.

Which of the following is the most common cause of cerebral palsy (CP)? A Perinatal asphyxia B Genetic abnormalities C Low birth weight/prematurity D Infection E Inborn error of metabolism

Answer C Low birth weight/prematurity Explanation Prematurity is the most common cause of CP (35%). Intrapartum asphyxia accounts for 9% of cases. Infection accounts for 15% of total spastic CP cases in normal weight children. Chromosomal/genetic and metabolic etiologies each account for 5%.

You are asked to evaluate an 8-year-old girl who presents with decreased urine output and suprapubic pain. These symptoms developed over a 2-3 week period. She has also complained of lower back pain that has been intermittent over the preceding 8-10 months but has become increasingly symptomatic. Her family denies any trauma, but the patient has recently developed some difficulty with walking (fatiguing more easily and stumbling). On examination, she is noted to have significant suprapubic fullness and discomfort, which is greatly relieved when a catheter is placed and drains 350 mL of urine. She has good peripheral strength but complains of back pain when straight-leg raising is attempted. Babinski sign is equivocal bilaterally. There is decreased tone of the rectal sphincter. She has no scoliosis, but a dimple with a small patch of hair is noted in the midline of the back at the L3 level. Which of the following is most likely to establish the reason for this patient's symptoms? A Cystoscopy B Renal ultrasound C MRI of the lumbar spine D Spine films E Intravenous pyelogram

Answer C MRI of the lumbar spine Explanation This presentation is consistent with a tethered spinal cord. She has evidence of increasing symptoms with walking, and her back pain and skin findings are suggestive of spinal dysraphism (spinal dimple with tuft of hair). The patient's urinary presentation is consistent with a neurogenic bladder. The presence of a full bladder that the patient has difficulty emptying indicates that the difficulty is below the level of the ureters. An MRI of the spine is most likely to demonstrate a condition that would cause a neurogenic bladder, back pain, neurologic findings, and the skin lesion over the lumbar area. Tethered cord syndrome (TCS) is a diverse clinical entity characterized by symptoms and signs caused by excessive tension on the spinal cord. The majority of cases are related to spinal dysraphism. TCS can present in any age group, and presentations differ according to the underlying pathologic condition and patient age, with pain, cutaneous signs, orthopedic deformities and neurological deficits being the most common. Surgical untethering is indicated in patients with progressive or new-onset symptomatology attributable to TCS. The surgical strategy aims to release the tethering structure and thus the chronic tension on the cord. Early operative intervention is associated with improved outcomes. The benefits of surgery are debated in asymptomatic patients and patients with normal imaging.

A 14-year-old boy presents for the new onset of right eye pain, swelling, and redness. He has intractable epilepsy and has been taking multiple antiseizure medications. Which of the following medications is most likely responsible for the patient's symptoms? A Phenytoin B Valproic acid C Topiramate D Lacosamide E Phenobarbital

Answer C Topiramate Explanation The patient's symptoms are consistent with acute-angle closure glaucoma, a known (possibly idiosyncratic) side effect of topiramate. In most cases symptoms resolve after cessation of topiramate, but ophthalmology should be involved for further management. Topiramate is widely used in the management of epilepsy and migraine prevention. Other side effects include somnolence, fatigue, psychomotor slowing, cognitive disorders, and nephrolithiasis.

A 2-week-old male infant with bilateral clubfeet presents with poor feeding. His mother has noted that he has great difficulty maintaining a strong suck during breastfeeding. On physical examination, his face appears expressionless, and he does not blink or grimace. The workup for congenital myasthenia is unremarkable. Which of the following findings is most likely to be identified during additional evaluation of this patient? A Heart defects B Bilateral choanal atresia C Urinary tract anomalies D 6th cranial nerve palsy E Bilateral retinal coloboma

Answer D 6th cranial nerve palsy Explanation The patient has clinical signs and symptoms consistent with Möbius syndrome, which is a rare congenital dysinnervation syndrome that results from hypoplasia or agenesis of the brainstem nuclei and muscle fibers. It is characterized by bilateral facial nerve (CN 7) paralysis. Abducens nerve (CN 6) paralysis and involvement of other cranial nerves can also be seen. Affected infants have an expressionless, dull appearance and are unable to smile, frown, suck, grimace, or blink. Associated anomalies include clubfeet, absent fingers and/or toes, and unilateral absence of the pectoralis muscle (Poland syndrome).

A 5-year-old child presents with nighttime episodes of facial twitching and difficulty talking. The episodes evolve into generalized shaking at times. He is developmentally normal. Which of the following is the pattern you would expect to see on EEG? A Hypsarrhythmia B Occipital sharp waves C 3 Hz generalized spike and wave D Centrotemporal sharp waves E 4-6 Hz polyspike and wave

Answer D Centrotemporal sharp waves Explanation This child has a history suggestive of benign rolandic epilepsy of childhood (a.k.a. benign epilepsy with centrotemporal spikes [BECTS]). The typical EEG pattern is that of centrotemporal sharp waves on the left and the right. The majority of children will outgrow their seizures by puberty. Many clinicians do not treat BECTS with antiseizure drugs if the episodes are infrequent and occur only at night. 3 Hz generalized spike and wave discharges are associated with absence seizures and typically manifest as staring episodes with behavioral arrest. 4-6 Hz polyspike and wave are associated with idiopathic generalized epilepsy with generalized tonic-clonic seizures. Occipital sharp waves are a finding in epilepsy with visual phenomena. Hypsarrhythmia is an EEG finding associated with infantile spasms.

The parents of a 6-year-old boy return for follow-up 2 weeks after their son began taking methylphenidate for attention-deficit/hyperactivity disorder (ADHD). His grades continue to be poor, and his teacher continues to be concerned that he is often unable to stay on task, is easily distracted, and, at frequent times throughout the day, "just stares off into space," seemingly unaware of his surroundings for seconds at a time per episode. He is sometimes noted to have fluttering of his eyelids. A change in postural tone may be noted during an episode. Which of the following is most likely to cause the onset of this patient's repetitive behaviors? A Prolonged, vigorous exercise B Valsalva maneuver C A diet high in carbohydrates and simple sugars D Hyperventilation for 3 minutes E Fasting for at least 12 hours

Answer D Hyperventilation for 3 minutes Explanation The patient is described as not responding to appropriate treatment of attention-deficit/hyperactivity disorder (ADHD). Although some children with ADHD require several therapeutic trials of medication before an improvement in symptoms is noted, lack of response to medical intervention should prompt an even more thorough investigation for other disorders that may mimic ADHD. For example, he has clinical signs and symptoms consistent with absence seizures (a.k.a. petit mal seizures), which are typically characterized by frequent brief episodes of staring and unresponsiveness, often associated with ≥ 1 automatisms. Hyperventilation will often provoke an absence seizure in children. Some patients may experience hundreds of seizures per day. If the brief seizures go unrecognized, an attention-deficit disorder may initially be suspected as the cause of a child's poor school performance. Lapses of awareness interfere with day-to-day activities at school and home, often frustrating the child, parents, and teachers and interfering with attainment of social skills, developmental milestones, and academic performance.

The parents of a 9-year-old boy bring their son to the emergency department because "something is wrong with his face." There is no history of trauma or associated systemic symptoms. On physical examination, he is unable to wrinkle his forehead on the left and has also lost the ability to close his left eye and raise the left corner of his mouth. Decreased nasolabial fold prominence on the left is also noted. Which of the following is the cause of this patient's clinical findings? A Upper motor neuron dysfunction of the right 7th cranial nerve B Upper motor neuron dysfunction of the right 5th cranial nerve C Lower motor neuron dysfunction of the left 5th and 7th cranial nerves D Lower motor neuron dysfunction of the left 7th cranial nerve E Lower motor neuron dysfunction of the left 5th cranial nerve

Answer D Lower motor neuron dysfunction of the left 7th cranial nerve Explanation The distribution of the symptoms indicates the dysfunction is of the 7th cranial nerve (CN 7, facial nerve) rather than the 5th cranial nerve (CN 5, trigeminal nerve). This patient has a lower motor neuron dysfunction of the left 7th cranial nerve. The pattern of innervation of the 7th cranial nerve has important clinical implications. The right and left upper motor neurons each innervate both the right and left lower motor neurons, which allow the forehead to move up and down. The lower motor neurons, which control the muscles of the lower face, are innervated only by the upper motor neuron from the opposite side of the cerebral cortex; therefore, with a unilateral upper motor neuron lesion, the upper part of the face will be spared due to the bilateral supply. In contrast, lower (peripheral) motor neuron dysfunction is characterized by patients being unable to wrinkle the forehead, close the eye, or raise the corner of the mouth on the affected side. Function on the contralateral side is normal.

You are called to the delivery suite to attend a newborn who has been experiencing difficulties with respirations and is demonstrating abnormal tone. The child was delivered precipitously by vaginal delivery to a mother who received no prenatal care. On examination, the child has poor general tone and is requiring flow-by oxygen and intermittent bagging to maintain oxygen saturation. The airway appears patent, but on examination of the oropharynx, there are prominent fasciculations of the tongue. The child is breathing with paradoxical retractions of the intercostal muscles on inspiration. The child also has diminished muscle tone in both upper and lower extremities. Which of the following is the most likely diagnosis? A Hypoxic-ischemic brain injury B Infantile botulism C Congenital muscular dystrophy D Spinal muscular atrophy Type 1 E Myasthenia gravis

Answer D Spinal muscular atrophy Type 1 Explanation Spinal muscular atrophy Type 1 (SMA1; a.k.a. Werdnig-Hoffmann disease or severe infantile SMA) arises from atrophy of the anterior horn cells in the spinal cord, leading to muscle and peripheral nerve atrophy. The disease presents between birth and 6 months of age. The child usually has marked hypotonia, absent tendon reflexes, and weakness of the intercostal musculature. Fasciculations of the tongue are a very common feature. Mental function is typically preserved in these children. Infantile botulism is caused by Clostridium botulinum spores in the child's gut that produce toxin. This causes constipation and a descending paralysis, leading to poor suck and swallow followed by respiratory compromise. The examination typically reveals a floppy, afebrile child with dilated pupils who is sluggish to react and has a dysconjugate gaze, swallowing difficulties, dry mucous membranes, and, in later stages, respiratory failure. Each of the remaining diseases presents with weakness and poor tone, but they lack the fasciculations of the tongue that are a feature linked to SMA1: Myasthenia gravis in the neonatal period occurs either from the transfer of antiacetylcholine antibodies from an affected mother or through an inherited defect in the acetylcholine receptors of the child. The 1st condition is transient because the antibodies are progressively lost. The 2nd defect is a lifelong condition. The child is usually hypotonic, with poor suck and diminished respiratory effort. Congenital muscular dystrophy arises from multiple genetic defects, but the group commonly presents with hypotonia, difficulty feeding, and respiratory weakness. Hypoxic-ischemic brain injury in a neonate can present with symptoms of encephalopathy, including lethargy, reduced spontaneous movements, poor tone, and respiratory difficulty. There can be posturing, seizures, and absent primitive reflexes but typically not fasciculations of the tongue.

The mother of a 15-year-old girl brings her daughter to the clinic to be evaluated. Despite attending school daily, she frequently misses classes and reports to the school nurse 1-2 times per week with complaints of sparkling and blurry vision followed by an upset stomach and headache. She cries until her mother has been called and then sleeps until her mother arrives, typically 1 hour later. She prefers a darkened, quiet room, and, upon occasion, has complained of nausea and vomited. The patient's mother states that the event tends to resolve with rest 3-4 hours after onset. This has been going on for several months. Complete evaluation, including an MRI of the brain and a thorough eye examination, reveals no abnormal findings. Which of the following would most likely resolve this patient's symptoms? A Psychiatric referral B Methylphenidate C Ophthalmology referral D Topiramate E Spinal tap with opening pressures

Answer D Topiramate Explanation Absences from school can occur for multiple reasons, including social and behavioral reasons. When a child begins to demonstrate such a pattern, it is prudent to evaluate for a pathological basis for the disturbance. Attributing the source to a behavioral cause without a thoughtful medical evaluation is not justified. The overall presentation is consistent with migraine headache with aura, and, with the frequency of symptoms, prophylactic agents such as topiramate, propranolol, cyproheptadine, or amitriptyline may be of benefit. Topiramate is approved for prevention of migraines in children 12-17 years of age. In 2017, results from the CHAMP (Childhood and Adolescent Migraine Prevention) trial showed no significant differences in reduction in headache frequency or headache-related disability in childhood and adolescent migraine with amitriptyline, topiramate, or placebo over a period of 24 weeks. The active drugs were associated with higher rates of adverse events. This study has led to the discussion about role of prophylactic agents versus placebo. However, until further studies are available or there are changes in guidelines from national committees, prophylactic medications remain a reasonable treatment option for children and adolescents with migraine.

During a well-child examination, a 2-month-old girl is noted to have several episodes of rhythmic flexor-extensor episodes lasting 2-3 seconds. These are characterized by sudden flexion of the neck, trunk, and arms and extension of the legs. Her parents report that they have witnessed similar episodes at home and that she will sometimes "scream just after she stops." Since the onset of the spells, her parents notice that she seems to be having a harder time with head control and is no longer smiling socially. Which of the following is the most appropriate initial treatment in this patient? A Ethosuximide B Intravenous immunoglobulin C Folic acid and vitamin B12 D Phenobarbital E Adrenocorticotropic hormone

Answer E Adrenocorticotropic hormone Explanation This patient has clinical signs and symptoms of infantile spasms, characterized by sudden, brief (1-3 seconds) contractions (simultaneous flexion of neck and trunk, with flexion and adduction of their extremities). The onset of infantile spasms is often associated with developmental regression, as is seen in this patient. Adrenocorticotropic hormone (ACTH; a.k.a. corticotropin) and vigabatrin are considered 1st line treatments for infantile spasms. In patients with infantile spasms due to tuberous sclerosis, vigabatrin is the drug of choice. A variety of antiseizure drugs have been used in affected children who do not respond to ACTH or vigabatrin—but usually with only partial, if any, improvement in symptoms. Pyridoxine (vitamin B6) is sometimes effective. Hypsarrhythmia is the characteristic interictal EEG pattern in patients with infantile spasms and consists of chaotic, disorganized, high-voltage polymorphic delta and theta rhythms with superimposed multifocal spike and wave discharges. Although clinical spasms resolve with age, overall prognosis is poor, with rates of intellectual disabilities as high as 80-90%. Many patients develop other seizure disorders later in childhood and adolescence, especially Lennox-Gastaut syndrome. Successful treatment of infantile spasms possibly improves the long-term prognosis. Phenobarbital has shown no benefit in cases of infantile spasms. It​ is used in infants who have idiopathic generalized tonic-clonic seizures​. Ethosuximide use is limited to absence seizures and has not been shown to have benefit in infantile spasms. Intravenous immunoglobulin has been studied in a limited way for Rasmussen encephalitis, characterized by progressive focal seizures and progressive hemiplegia. It has not shown benefit in infantile spasms. Neither folic acid nor vitamin B12 have antiseizure properties. Folic acid supplementation is recommended in females of childbearing age who take antiseizure medication to reduce the risk of neural tube defects.

A 6-year old child is brought by his parents to the office with concerns that the child has not been able to start school. He appears to them to have "weak muscles" that have resulted in a "funny way of walking," as well as frequent falls and difficulty in rising from a chair or ascending the stairs at bedtime. They describe a curious method of rising from the floor wherein he "walks up" his torso (using a hands-to-knees sequence which you recognize as Gowers sign). On your examination, you are struck by the apparent hypertrophy of his calf muscles. What is the most likely diagnosis? A Becker muscular dystrophy B Late-onset Pompe disease C McArdle disease D Dermatomyositis E Duchenne muscular dystrophy

Answer E Duchenne muscular dystrophy Explanation The most likely diagnosis is Duchenne muscular dystrophy (DMD), which is an X-linked recessive disease. Motor symptoms usually become noticeable by 5 years of age, and loss of ambulation occurs between 7 and 13 years of age. Patients with DMD have relentless chronic progression of cardiopulmonary disease, including arrhythmias and conduction system disease, accompanying their characteristic cardiomyopathy. Creatine kinase (CK) levels are usually elevated. There is an associated risk of sudden cardiac death. Dermatomyositis (DM) is a systemic connective tissue disease that may effect skin, muscles, and multiple other organs. Juvenile DM usually presents at a later age. Becker muscular dystrophy presents later in life and is not as severe as DMD. McArdle disease and late-onset Pompe disease are disorders of muscle glycogen metabolism. These patients usually present with muscle cramps and fatigue after exercise.

A 16-year-old girl presents with complaints of headache, which has been gradually worsening over the preceding 2-3 months. She states that the headaches are dull in nature and centered over the top of her head. They initially responded to acetaminophen and subsequently to ibuprofen, but she is concerned about taking pills every day. On physical examination, she is alert and oriented. She has normal ocular movements with a small degree of vertical nystagmus. There is no papilledema. Cranial nerves are otherwise intact. There is no temporomandibular joint (TMJ) pain or facial tenderness. Tympanic membranes are within normal limits, and dentition is unremarkable. Neurologic examination is otherwise unremarkable. Which of the following is the most appropriate intervention at this time? A Refer for eye examination. B Recommend a headache diary. C Prescribe prophylactic propranolol. D Continue ibuprofen as needed.

Answer E Obtain MRI of the brain. Explanation Headaches are caused by many processes, and the ability to identify the etiology is important in the treatment of this complaint. Certain elements of the history and physical examination are "red flag" items that prompt an expedited evaluation. Recent onset and increasing severity and frequency of the headaches is of concern, as is vertical nystagmus, which should always be considered an abnormal finding. It is associated with demyelinating diseases, tumors, and increased intracranial pressure. With this finding, the most appropriate intervention at this time would be MRI of the brain. It has the advantage over CT scan for identifying demyelination. Indications for Head Imaging: These include auras lasting > 1 hour, persistent neurologic findings, abnormal neurologic examination between headaches, predominantly occipital headaches, decline in cognitive function, decline in growth velocity, and recent significant change in headache pattern. There are some red flags that should be taken seriously (see below), and the threshold of doing imaging in these patients should be low. Red Flags for Headache: A common mnemonic used for headache red flags is SNOOP: systemic symptoms (e.g., fever, chills, weight loss) or secondary headache risk factors (e.g., HIV, cancer); neurologic symptoms or signs (e.g., confusion, impaired consciousness, focal findings); older (e.g., new-onset or progressive headache, especially > 50 years of age—temporal arteritis); onset (e.g., sudden, abrupt, thunderclap); and previous headache history (e.g., change in frequency, severity, clinical features). Referral for an eye examination may be considered in the workup for headaches; however, in this case, the red flags for a possible secondary cause of headache prompt immediate neuroimaging with an MRI. Prophylactic propranolol is a consideration in the case of migraine headache, a primary headache disorder; however, the vertical nystagmus suggests an alternative diagnosis that merits further investigation. A headache diary is often recommended in the ongoing management of migraine and other headache disorders but is not part of the initial workup. Ibuprofen is indicated for the acute symptomatic relief of some headache disorders; however, the presence of the nystagmus red flag prompts further investigation for a secondary cause of the headaches.

The parents of an 11-year-old boy report that they were startled by the sound of their son "choking and making loud noises." When they checked on him, they noted that his face was "twitching" and that he was drooling. During the event, he was having great difficulty speaking and was only able to "grunt" in response to their questions. Subsequent evaluation includes a negative urine drug screen, a normal cranial MRI, and an EEG with centrotemporal spike and wave discharges and normal background activity. Which of the following statements best describes associated characteristics of patients with this type of seizure disorder? A Associated clinical findings often include multiple café-au-lait spots. B Monotherapy with an antiseizure medication is rarely successful in decreasing the frequency of seizure episodes. C The majority of affected patients exhibit moderate-to-severe intellectual disabilities. D Secondary generalized tonic-clonic seizures do not occur. E The majority of episodes occur during sleep or upon awakening.

Answer E The majority of episodes occur during sleep or upon awakening. Explanation The clinical symptoms presented describe a focal seizure without impaired consciousness (formerly called simple partial seizures) and with motor symptoms involving the face. Associated EEG findings of centrotemporal sharp peaks in the presence of normal background activity are diagnostic for benign epilepsy with centrotemporal spikes (BECTS; a.k.a. benign rolandic epilepsy of childhood), which accounts for 10-20% of all childhood epilepsies. Seizure activity is limited to the face, oropharyngeal musculature, and, at times, the upper extremities. Associated symptoms include facial numbness or twitching, guttural or "grunting" vocalizations, hypersalivation, drooling, and dysphasia. The majority (~ 75%) of seizures occur during the night or upon awakening; therefore, many are clinically unrecognized. Secondary nighttime generalized tonic-clonic seizures occur in upwards of 50% of patients and are often the first indication of the underlying seizure disorder. Prognosis is excellent, with a mean duration of epilepsy of 2 years and spontaneous remission in the majority of patients before 12-13 years of age. Peak onset is between 7 and 9 years of age. Monotherapy is usually effective. Treatment is optional and is associated with a reduction in generalized but not partial seizures. Some individuals with rolandic epilepsy of childhood may have mild cognitive impairment.

You see a 1-year-old female whose problems began in infancy. Initially, she developed hypotonia. Now she has progressed to the point where she has severe weakness. She has hyporeflexia on examination and muscle fasciculations are noted—particularly of her tongue. She has not had any seizures or fever. Which of the following is the most likely diagnosis? A Wilson disease B AIDS C Medulloblastoma D Gaucher disease E Werdnig-Hoffmann disease

Answer E Werdnig-Hoffmann disease Explanation Werdnig-Hoffmann disease, or spinal muscular atrophy Type 1 (SMA1) constitutes over 80% of cases of spinal muscular atrophy. Suspect it in a child less than 2 years of age who has progressive weakness with tongue fasciculations. Infants with SMA1 experience weakness with onset prior to 6 months of age and never achieve the ability to sit independently. Muscle weakness, lack of motor development, and poor muscle tone are the major clinical manifestations. Cognitive development is normal. Most children with SMA1 die by 2 years of age.

A 4-year-old boy presents for an initial health maintenance examination after being adopted from Guatemala. On physical examination, he is noted to have a short neck, low hair line, and limited range of motion of his head and neck. Anteroposterior, lateral, and oblique views of the cervical spine show evidence of multiple fused cervical vertebrae. Which of the following congenital orthopedic abnormalities is a commonly associated finding in patients with this disorder? A Talipes equinovarus B Failure of the scapula to descend to its normal position C Genu varum D Developmental dysplasia of the hip E Absent radius

B Failure of the scapula to descend to its normal position Explanation The patient in the vignette has clinical findings consistent with Klippel-Feil syndrome, characterized by congenitally fused cervical vertebrae that may result in a short neck, limited range of motion of the neck, and the appearance of a low hairline at the back of the head. The shortened neck can lead to torticollis and/or facial asymmetry. Some children also have associated neural tube defects and neurological deficits. Associated orthopedic abnormalities include Sprengel deformity (or failure of the scapula to descend to its normal position) and also atlantoaxial instability, hemivertebrae, and scoliosis. Individuals with Klippel-Feil syndrome should undergo imaging of the genitourinary tract, because up to 40% may have associated renal and/or collecting system abnormalities. Additional associated defects include congenital heart disease, particularly ventricular septal defect, conductive or neural hearing loss, cleft lip, and intellectual disabilities.

Parents of a 3-year-old boy bring their son to the clinic with concerns of the recent development of episodic unsteadiness and dizziness without warning. They state that he seems to lose his balance, is very unsteady with his walking, and tries to cling to them. This will last for 3-4 minutes and resolves spontaneously. He seems to be completely alert before, during, and after each episode. They deny any recent illnesses, and his past history and development have been unremarkable. His physical examination, including neurologic examination, is completely normal. Which of the following is the most likely to be true in this case? A A sleep-deprived EEG will show abnormal focal discharges. B The child is more likely to develop migraine headaches in the future. C Antiseizure medications are often required to suppress these symptoms. D Cold caloric testing demonstrates an exaggerated vestibular response. E Benign paroxysmal positional vertigo is the most common cause of these symptoms at this age.

B The child is more likely to develop migraine headaches in the future. Explanation Benign paroxysmal vertigo of childhood (BPVC) is a self-limited condition (though symptoms may last 2-3 years) that causes sudden episodes of vertigo in children, usually between 1 and 4 years of age. The attacks last a few minutes and can be distinguished from seizures by the maintenance of alertness during the episode. Benign paroxysmal vertigo is thought to be an early childhood periodic syndrome that may be a precursor to migraine. Therefore, an emphasis on childhood migraine and migraine history in the family are important in order to make the proper diagnosis. The management of vertigo attacks requires attention to the family's migraine triggers and to lifestyle changes. BPVC is a perfectly known entity that must be distinguished from benign paroxysmal positional vertigo (BPPV), which manifests as brief attacks of vertigo triggered by changes in the position of the head (e.g., while turning in bed or lying down). BPPV is thought to be caused by free-floating particles leaving the macula of the utriculus and entering one of the semicircular canals—usually the posterior canal or, more rarely, the horizontal or superior canals. Diphenhydramine may be useful to suppress symptoms such as nausea and vomiting caused by vertigo (both BPVC and BPPV), particularly when episodes are frequent or severe.

An 18-year-old female, who is at 24-weeks of gestation, is found to have an elevated level of serum α-fetoprotein. An ultrasound reveals the presence of a meningomyelocele. Which of the following medications, when taken during pregnancy, is most likely to be associated with these findings?

Valproic acid.

A previously healthy 8-year-old girl presents to the emergency department with an unprovoked generalized tonic-clonic seizure lasting 20 minutes. She has never had a seizure or head injury before and there is no family history of seizures. Her physical examination shows a mildly sleepy girl, with no other neurological abnormalities. Electrolytes, glucose, and a complete blood cell count are normal. Magnetic resonance imaging of the head is also normal. She is admitted for observation and the next day has an electroencephalogram that is normal. You are planning to discharge her home. Of the following, the BEST statement about her seizure recurrence risk is it Incorrect Correct Checked A. is 25% to 45% IncorrectCorrectChecked B. is greater than 45% because she had a prolonged seizure IncorrectCorrectChecked C. is greater than 45% because she is prepubertal IncorrectCorrectChecked D. will stay the same over the next 2 years, then decrease IncorrectCorrectChecked E. will stay the same over the next 14 years, then decrease

Between 40% to 50% of adults and children with a first, unprovoked seizure will have a recurrent seizure. Factors that increase the risk of recurrent seizure include abnormal findings on electroencephalogram and abnormal results from magnetic resonance imaging of the brain, such as remote brain injury or brain malformation (which may not be seen on computed tomography). A prolonged first seizure does not increase the risk of recurrent seizure. In a typically developing child with a first, unprovoked seizure whose electroencephalogram and magnetic resonance imaging of the brain are normal, the recurrence risk is as low as 25%. For the girl in the vignette, the seizure recurrence risk is 25% to 45%; it will be at the lower end of the range if her magnetic resonance imaging results are normal. About half of recurrent seizures occur in the first 6 months after the first seizure, and almost 90% of recurrent seizures happen in the first 2 years. Identifying an etiology for the seizure can help predict the risk of recurrence. Seizures and epilepsy in childhood can be due to an epilepsy syndrome or to an underlying etiology. Underlying etiologies can be subdivided into 6 categories: genetic, structural, metabolic, immune, infectious, and unknown. This classification scheme is currently being updated by the International League Against Epilepsy (www.epilepsydiagnosis.org). Examples of epilepsy syndromes include childhood absence epilepsy and juvenile myoclonic epilepsy. Examples of underlying etiologies of epilepsy include traumatic brain injury, mitochondrial disorders and genetic disorders. In a particular child, more than 1 epilepsy syndrome or etiology may apply. For example, a child younger than 4 years of age who develops absence seizures may also have glucose transporter deficiency, a metabolic and genetic cause for epilepsy; or, a child with tuberous sclerosis has both a genetic and structural cause for epilepsy. Clinicians should be aware of the increased risk of developing seizures in children with one or more underlying etiologies, but it is rarely necessary to start anticonvulsants prior to the onset of clinical seizures.

A mother comes in with her child who has been diagnosed with spina bifida. Which of the following would be the risk of recurrence in the next sibling, should she decide to have another child?

C 2-3% Explanation The risk of giving birth to a child with spina bifida or neural tube defect (NTD) is ~ 1/1,000 to 2/1,000 births (0.1%). The risk to subsequent siblings is 2-3%. If two siblings were previously affected, the risk is 10%. Maternal use of valproic acid increases risk to 1% (effectively increasing the risk 10-fold). The routine administration of 0.4 mg of folic acid in women of child bearing age prevents > 50% of NTDs. Administration of high dose folic acid (4 mg) before and during pregnancy in at-risk populations (women who have had a previous NTD-affected pregnancy or are on valproic acid or carbamazepine) can reduce risk or NTD-associated pregnancy by ~70%.

Difference between Chiari I and Chiari II malformation?

Chiari Type II malformation is commonly associated with myelomeningocele, but Chiari Type I malformation is less likely. Chiari Type I is the downward displacement of the cerebellar tonsils, and in severe cases the medulla, through the foramen magnum.

What is the characteristic EEG finding in infants with infantile spasms, and what is the type of seizure activity typical of this disorder?

Explanation Infantile spasms are characterized by hypsarrhythmia on EEG. Hypsarrhythmia is characterized by high-voltage, asynchronous, chaotic, slow wave and spike activity in all cortical areas. Typical seizure activity, which often follows arousal, includes spasms, characterized by a sudden flexion and adduction of the extremities, along with simultaneous flexion of the neck and trunk. Although an individual spasm is usually ≤ 10 seconds in duration, the majority of infantile spasms occur in clusters with several spasms during a single cluster. The majority of affected children present at < 1 year of age, with a peak incidence of onset between 3 and 7 months of age. Prognosis is poor, with only a small minority of patients able to achieve normal developmental milestones. Mortality rates as high as 30% have been reported. The mainstay of medical treatment of infantile spasms is therapy with adrenocorticotropic hormone (ACTH) or vigabatrin. For patients who have both infantile spasms and tuberous sclerosis, vigabatrin is the recommended treatment.

What studies are recommended for a patient with a hairy tuft overlying a 3-mm dimple with a visible base on the child's lumbar spine?

MRI of the spine should be obtained. A lumbosacral dimple with overlying tuft of hair is associated with tethered cord, syringomyelia, spina bifida occulta, or diastematomyelia. Imaging of the spine is indicated regardless of size of the dimple or if the base is visible, but MRI of the spine is more sensitive than U/S.

What are the criteria for tourette's?

Multiple motor and vocal tics lasting > 1 year In addition to the presence of multiple motor and vocal tics for a least 1 year, other criteria for the diagnosis of Tourette syndrome include no tic-free interval lasting > 3 months, onset prior to 18 years of age, and no underlying medical cause. Comorbid disorders associated with Tourette syndrome include attention-deficit/hyperactivity disorder and obsessive compulsive disorder. However, these are not prerequisites for diagnosis of Tourette's.

Define simple febrile seizure

Simple febrile seizures last < 15 minutes, are not recurrent in a 24-hour period, and are generalized in nature. Complex febrile seizures last ≥ 15 minutes, have > 1 episode in a 24-hour period, or demonstrate a focal symptomatology. It is estimated that ∼ 20-30% of all the febrile seizures may be complex febrile seizures. it is important to differentiate between the two, as risk of future epilepsy with simple febrile seizures is approximately 1-2% (not too different from the general population), but with complex febrile seizures is about 5-10%

An 11-year-old girl of Asian descent with a history of a seizure disorder presents to the emergency department with altered mental status and hemiparesis. A noncontrast CT scan of the head reveals blood in the basal ganglion, thalamus, and ventricular system. Additional evaluation with a cranial MRI reveals dilated collateral vessels in the basal ganglia and thalamus. Which of the following is most likely to be identified during additional evaluation of this patient? A Morbilliform rash associated with injected conjunctiva and swollen hands and feet. B A purplish-red, nonelevated vascular malformation in the trigeminal distribution, including the ophthalmic division. C Additional findings of hemispheric atrophy associated with serpiginous parenchymal calcifications on non-contrast CT. D Elevated levels of homocysteine in the blood and urine. E Stenotic lesions in the distal internal carotid arteries and arteries around the circle of Willis on angiography.

Stenotic lesions in the distal internal carotid arteries and arteries around the circle of Willis on angiography. Explanation The patient has findings consistent with moyamoya disease, characterized by bilateral stenosis or occlusion of the distal carotid arteries and the arteries around the circle of Willis. As a result, prominent arterial collateral vessels form and account for a hazy ("puff of smoke") angiographic appearance of the vascular collateral network ("moyamoya" is a Japanese word meaning "puff of smoke"). Moyamoya disease is more common in Asian patients, but moyamoya syndrome is a secondary vasculopathy that can be associated with neurofibromatosis Type 1, trisomy 21, sickle cell disease, and cranial irradiation. It may present with altered mental status, recurrent headaches, chorea, transient ischemic attacks, hemorrhagic stroke, and/or focal neurologic deficits. Demonstration of stenotic and/or occlusive lesions in the distal internal carotid arteries and the arteries around the circle of Willis on conventional or MR angiography is diagnostic.

You are seeing an 11-year-old girl in your office for a health supervision visit. She reports daily headaches that started 2 months ago. The headaches are described as pounding, with nausea and vomiting, but no phonophobia or photophobia. The pain worsens when she leans over to put on her shoes and socks. Occasionally, her vision will turn gray for a few seconds. When her headache is severe, the girl develops double vision. She does not take any medications or supplements. She reports that her weight has increased by more than 20 lbs since her visit 1 year ago. On physical examination, the girl's blood pressure is 102/78 mm Hg and her heart rate is 92 beats/min. Her body mass index is at the 96th percentile. Her neurologic examination shows an inability to abduct her right eye. Of the following, the MOST likely additional finding on this girl's physical examination would be A. cataracts B. nystagmus C. orbital bruit Correct D. papilledema E. ptosis and pupillary miosis

The girl in the vignette has pseudotumor cerebri. Her neurologic examination shows a right cranial nerve VI palsy, which is a sign of increased intracranial pressure. Papilledema may be seen with increased intracranial pressure, so of the choices, this is the most likely finding in this girl. Pseudotumor cerebri is a condition in which there is increased intracranial pressure but no intracranial mass, hydrocephalus, or other structural abnormality. It is sometimes called "idiopathic intracranial hypertension." Risk factors for pseudotumor cerebri include obesity, especially with recent weight gain, and female sex. Medications such as isotretinoin or doxycycline can cause pseudotumor cerebri as well. Symptoms of pseudotumor cerebri include headache, nausea, vomiting, transient visual obscurations (the entire visual field briefly turns gray, as in the girl in the vignette), tinnitus, and headache that worsens with bending over. Neurologic examination findings include papilledema and sometimes a cranial nerve VI palsy. Performing the fundoscopic examination in a darkened room makes it much easier to see papilledema. Untreated pseudotumor cerebri can result in permanent vision loss. In patients with new-onset headaches, clinicians should always assess for evidence of increased intracranial pressure. None of the other choices listed are associated with increased intracranial pressure. Cataracts are a clouding of the lens in the eye. Symptoms include slowly progressive blurry vision, not transient visual obscurations as described for the girl in the vignette. Nystagmus is a rapid beating movement of the eyes. It can be seen with injury to the brainstem or cerebellum, and sometimes as a medication side effect. It is not a typical finding in increased intracranial pressure, unless the increased pressure is due to a tumor or stroke affecting the brainstem or cerebellum. Orbital bruits, heard on auscultation with the stethoscope bell over the eye, are associated with vascular abnormalities such as carotid stenosis, arteriovenous fistula, or carotid cavernous fistula. The girl in the vignette does not have symptoms of an intracranial vascular abnormality, so this is not the best choice. Ptosis and pupillary miosis are 2 of the 3 findings in Horner syndrome, the third is anhidrosis. Horner syndrome can occur from disruption of the sympathetic pathway that innervates the eye, anywhere along its course from the brain, neck, chest and up to the orbit. It is not a sign of pseudotumor cerebri, and so would not be an expected finding in the girl in the vignette. PREP Pearls Untreated pseudotumor cerebri can result in permanent vision loss. Clinicians should always assess patients with headache for signs and symptoms of increased intracranial pressure. Signs of increased intracranial pressure include papilledema and cranial nerve VI palsy.

During the previous 24 hours, a 5-day-old infant has at least 7 multifocal seizures, each lasting ≤ 2 minutes. None of the seizures were associated with generalized tonic-clonic activity. Following a brief postictal state, the infant's physical examination has remained normal. He was born at 40 weeks of gestation following an uncomplicated pregnancy and delivery. Apgar scores were 9/9. Laboratory findings, in addition to cerebrospinal fluid analysis and neuroimaging, are all normal. Family history is negative for any seizure disorders. Which of the following statements best describes the prognosis in this patient? A The patient will require feedings with soybean-based formula. B Long-term prognosis is concerning for neurodevelopmental delays. C The patient will require lifelong supplementation of oral pyridoxine. D The patient will require antiseizure therapy for at least 2 years. E The patient will not experience any sequelae or require long-term antiseizure therapy.

The patient will not experience any sequelae or require long-term antiseizure therapy. Explanation The patient has historical and clinical findings consistent with benign neonatal seizures (formerly benign neonatal convulsions), and he will not experience any sequelae or require long-term antiseizure therapy. Affected infants are typically born at term with high Apgar scores following a normal pregnancy and delivery. The incidence of benign neonatal seizures peaks at 4-6 days of life (so have been referred to a "fifth day fits"). Seizures are most often tonic and may be unifocal or multifocal in nature. The neurologic examination before seizures and interictally is normal, as are the laboratory findings (including metabolic studies, neuroimaging, and cerebrospinal fluid analysis). Usually, there is not a family history of neonatal seizures or postneonatal epilepsy.

WHich medication is associated with hyperammonemic encephalopathy?

Valproic acid The disorder is usually temporally related to either onset of therapy or an increase in dose. Hyperammonemic encephalopathy is more common among patients on combination therapy for a seizure disorder. Pathogenesis of the disorder is likely related to overlapping metabolic pathways involving both ammonia and valproic acid.

What's the initial agent of choice for cluster headache in pediatrics?

Verapimil Cluster headache presents as severe, unilateral pain in and around the eye and temporal area. Pain is often described as similar to an "ice pick" or "hot poker." It is so severe that patients will typically move and/or rock constantly, unable to remain still. Pain rapidly peaks within minutes of onset, followed by resolution over the next several hours. Headache is accompanied by unilateral autonomic symptoms ipsilateral to the pain; signs and symptoms include miosis, ptosis, lacrimation, conjunctival injection, eyelid edema, sensation of fullness in the ear, nasal congestion, and rhinorrhea. Headaches are typically recurrent—sometimes multiple times a day over a period of 6-12 weeks. A period of remission, as long as 12 months, then follows. Administration of 100% oxygen is often effective as acute therapy aimed at aborting an individual attack. Triptans (intranasal or subcutaneous) are also used as first line treatment options. Verapamil is the drug of choice for initial prophylactic treatment of cluster headaches. If verapamil is ineffective or cannot be tolerated, alternatives include lithium, glucocorticoids, or topiramate. Fluoxetine and other selective serotonin reuptake inhibitors are not indicated in the acute or prophylactic treatment of cluster headaches. Propranolol, amitriptyline, and valproic acid are effective in some individuals for the prevention of migraine but not for cluster headaches.

Which vitamin deficiency can cause progressive ataxia?

Vitamin E Other signs and symptoms of vitamin E deficiency include hyporeflexia, abnormal vibratory sensation and proprioception, limitation of eye movement, and muscle weakness. Ataxia is characterized by a wide-based, swaying gait. There are many causes of ataxia in children; most cases present acutely, with rapid onset of symptoms. The most common cause is acute cerebellar ataxia, a syndrome thought to be associated with recent viral infection, particularly varicella. Other causes of acute ataxia include toxic ingestions (eg, alcohol, benzodiazepines, anticonvulsants), acute infections (eg, viral encephalitis), postinfectious (eg, acute cerebellitis), brain mass, or stroke. Some metabolic neurodegenerative conditions (eg, maple syrup urine disease) exhibit an intermittent ataxia. More progressive ataxias are often the result of inherited conditions (eg, Friedreich ataxia, ataxia-telangiectasia [AT]). Vitamin E deficiency can lead to progressive ataxia. This deficiency usually results from malabsorption or a mutation in the alpha-tocopherol transfer protein gene; it is rarely caused by poor diet. Ataxia from vitamin E deficiency can be treated with the administration of large doses of vitamin E, or by addressing the reason for the malabsorption. Many other causes of progressive ataxia do not have available treatments. All cases of ataxia warrant a diagnostic evaluation. Acute cases should be evaluated emergently, beginning with a careful history and physical examination. Although acute cerebellar ataxia is the most common cause, it is a diagnosis of exclusion that can be made if tests reveal no more serious and treatable causes. It is self-resolving. Computed tomography of the head can identify brain tumors, and is indicated in patients also presenting with focal neurologic findings, altered consciousness, or new-onset headaches. Toxicology screening tests and drug level measurement can detect ingestions or excessive doses of medications. The cerebrospinal fluid should be evaluated in patients presenting with fever, meningismus, seizures, or altered sensorium to identify meningitis, cerebellitis, or encephalitis. Neuroblastoma can present with ataxia; if suspected, especially in patients with opsoclonus-myoclonus, urine catecholamines should be measured. Cases presenting with intermittent or progressive ataxia warrant evaluation by a neurologist, but are typically caused by genetic conditions, including mitochondrial disorders or enzymatic defects. Among these, Friedreich ataxia is the most common, typically presenting in adolescence with progressive ataxia in all extremities, diminished tendon reflexes, and lower extremity weakness. It is diagnosed clinically and confirmed with genetic testing. AT presents earlier, typically in toddlerhood, often with abnormal eye movements and oculocutaneous telangiectasias. AT is associated with an elevated serum alpha-fetoprotein level. The diagnosis can be confirmed with genetic testing. The girl in the vignette does not have typical eye and skin findings of AT, making vitamin E deficiency, although rare, more likely in her case. It is important to investigate this cause of progressive ataxia, because treatment with vitamin E can slow the progression of neurologic symptoms associated with the disorder.

What is the recurrence risk of a seizure in a patient whose EEG and MRI are normal?

•Seizure recurrence risk is as low as 25% in a typically developing child with a first, unprovoked seizure whose electroencephalogram and magnetic resonance imaging of the brain are normal


Kaugnay na mga set ng pag-aaral

Chemistry Quiz 4 Review Questions

View Set

Foundations and Practice of Mental Health Nursing

View Set

U.S. History Chapter 6 The Progressive Movement

View Set